If Martin lectures at 1:00, then any of the following could be true EXCEPT:

Abigail on February 26, 2022

Why is a incorrect?

Hi, I don't understand why a is incorrect. I set up a couple of scenarios. Th: J, O/K, K/O F: L, M/N, N/M For this question the first scenario doesn't work because if M is at 1 then J automatically takes the other 1st position, meaning L cannot be 1st because he won't be in Friday. Which would put me in the scenarios below. Th: M, N, L/O F: J, K, O/L or Th: M, L/N, N/L F: J, O/K, K/O The question essentially asked what must be false. So, I chose A because J cannot be Thursday is M is at 1st. Can someone please tell me how my reasoning is wrong? Thanks!

Reply
Create a free account to read and take part in forum discussions.

Already have an account? log in

Emil on March 9, 2022

Hi Abigail,

I think we could put J on thursday and M on friday, both at 1PM, as long as L lectures on thursday instead of friday. L only has to b at 1 if L is on friday, we do not know when L lectures if on thursday.

So, we have a valid scenario as follows, with the order being 1,2,3.

Th: M, L/N, L/N
Fr: J, K/O, K/O

However, since we know that M and J both lecture at, we know that L cannot lecture on friday according to the contrapositive of rule 4.